Solving differential equation using Laplace Transform

Click For Summary
The discussion focuses on solving the differential equation y'' + 4y' + 4y = t^2 e^{-2t} using Laplace transforms, with initial conditions y(0) = 0 and y'(0) = 0. The Laplace transform is applied to the entire equation, leading to the expression (p^2 + 4p + 4)Y = 2/(p+2)^3. The solution for Y is derived as Y = 2/(p+2)^5, and the inverse Laplace transform is sought to find y. The confusion arises in correctly applying the inverse transform, specifically in relating it to the expression 2t^4 e^{-2t}/4!, which is clarified through the use of specific Laplace transform properties and formulas. The solution process demonstrates the application of Laplace transforms in solving linear differential equations effectively.
Edge Of Pain
Messages
21
Reaction score
0

Homework Statement


solve the following differential equation using Laplace transforms:
y'' + 4y' + 4y = t^2 e^{-2t}, y_0 = 0, y'_0 = 0

y_0 and y'_0 are initial conditions.

Homework Equations


Using L to represent the Laplace transform, we have that

<br /> L(y) = Y<br />
<br /> L(y&#039;) = pY - y_0<br />
<br /> L(y&#039;&#039;) = p^2 Y - py_0 - y&#039;_0<br />

The Attempt at a Solution


Taking the Laplace transform of the entire DE gives

<br /> p^2 Y - py_0 - y&#039;_0 + 4pY - 4y_0 + 4Y = L(t^2 e^{-2t})<br />

From Laplace transform tables (using M. Boas, page 469) we have

<br /> L(t^2 e^{-2t}) = 2/(p+2)^3<br />
<br /> ∴ p^2 Y - 0 -0 +4pY - 0 +4Y = 2/(p+3)^3<br />
<br /> ∴ (p^2 + 4p + 4)Y = 2/(p+2)^3<br />
<br /> ∴Y = 2/(p+2)^5<br />

<br /> ∴ y = L^{-1}(Y) = L^{-1}(2/(p+2)^5)<br />

Then from tables (pg 469, M Boas, mathematical methods for the physics sciences), we use the following relation:
<br /> L(t^k e^{-at}, k&gt;-1<br />

becomes

<br /> k!/(p+a)^{k+1}<br />

This all makes sense and I'm confident that this is correct so far

But somehow this means that the inverse Laplace transform of 2/(p+2)^5 is 2t^4 e^{-2t}/4! (which is also the solution to the DE - since it's L^{-1}(Y) = y). I don't see how to make this jump and I don't see how that is correct.
 
Physics news on Phys.org
##s## is more commonly used for the transform variable. Use $$
\mathcal L^{-1}F(p+a) = e^{-at}\mathcal L^{-1}F(p)$$where in your problem ##F(p) = \frac 2 {p^5}##. You need to multiply numerator and denominator by ##4!## to make the formula fit.
 
Last edited:
  • Like
Likes Edge Of Pain
that is just a particular case of
$$L\left(C\dfrac{t^k}{k!}e^{-a \, t}\right)=C(p+a)^{-(k+1)}$$
which we can get by combining
$$L\left(C\dfrac{t^k}{k!}\mathrm{f}(t)\right)=\dfrac{C}{k!}\left(-\dfrac{d}{dp}\right)^k L(\mathrm{f}(t))\\
L\left(e^{-a \, t}\right)=(p+a)^{-1}$$
into
$$L\left(C\dfrac{t^k}{k!}e^{-a \, t}\right)=\dfrac{C}{k!}\left(-\dfrac{d}{dp}\right)^k L(e^{-a \, t})=\dfrac{C}{k!}\left(-\dfrac{d}{dp}\right)^k (p+a)^{-1}=C(p+a)^{-(k+1)}$$
 
  • Like
Likes Edge Of Pain
Question: A clock's minute hand has length 4 and its hour hand has length 3. What is the distance between the tips at the moment when it is increasing most rapidly?(Putnam Exam Question) Answer: Making assumption that both the hands moves at constant angular velocities, the answer is ## \sqrt{7} .## But don't you think this assumption is somewhat doubtful and wrong?

Similar threads

  • · Replies 1 ·
Replies
1
Views
1K
  • · Replies 7 ·
Replies
7
Views
2K
  • · Replies 1 ·
Replies
1
Views
1K
  • · Replies 10 ·
Replies
10
Views
2K
Replies
5
Views
2K
Replies
2
Views
2K
Replies
3
Views
2K
  • · Replies 4 ·
Replies
4
Views
3K
  • · Replies 2 ·
Replies
2
Views
2K
Replies
2
Views
2K